Đến nội dung

Hình ảnh

$\sum \frac{ab}{3+bc}\leq\frac{3}{4}$

* * * - - 2 Bình chọn

  • Please log in to reply
Chủ đề này có 3 trả lời

#1
moonkey01

moonkey01

    Hạ sĩ

  • Thành viên
  • 50 Bài viết

Cho $a,b,c\geq 0$ thoả mãn $a+b+c=3$. Chứng minh rằng $\sum \frac{ab}{3+bc}\leq\frac{3}{4}$



#2
hantai

hantai

    Lính mới

  • Thành viên mới
  • 8 Bài viết

vừa tham gia trường đông ak bạn



#3
Minhnguyenthe333

Minhnguyenthe333

    Trung úy

  • Thành viên
  • 804 Bài viết

Cho $a,b,c\geq 0$ thoả mãn $a+b+c=3$. Chứng minh rằng $\sum \frac{ab}{3+bc}\leq\frac{3}{4}$

$VT=\frac{abc(a^2b+b^2c+c^2a)+9(ab+bc+ca)+3abc+3(a^2b^2+b^2c^2+c^2a^2)}{a^2b^2c^2+9abc+9(ab+bc+ca)+27}$

+Áp dụng bđt Cauchy: $\sum_{cyclic} (a^3+a^2b+ab^2)\geqslant 3(a^2b+b^2+c^2a)\iff a^2+b^2+c^2\geqslant a^2b+b^2c+c^2a$

 

+Đổi biến $p,q,r$ thì ta được: $VT\leqslant \frac{3q^2-2qr+9q}{r^2+9r+9q+27}$

Do đó ta chỉ cần chứng minh $\frac{3q^2-2qr+9q}{r^2+9r+9q+27}\leqslant \frac{3}{4}\iff 3r^2+r(8q+27)-(12q^2+9q-81)\geqslant 0$ $(*)$

Ta xét 2 trường hợp:
$q\leqslant \frac{9}{4}:$ Theo Schur bậc 1 thì $r\geqslant \max\{0,\frac{p(4q-p^2)}{9}\}=\max\{0,\frac{4q-9}{3}\}=0$

Thế thì $(*)\geqslant (q+3)(\frac{9}{4}-q)\geqslant 0$

$\frac{9}{4}\leqslant q\leqslant 3:$ Theo Schur bậc 2 thì $r\geqslant \frac{(4q-p^2)(p^2-q)}{6p}=\frac{(4q-9)(9-q)}{18}$

Do đó $(*)\geqslant (q-3)(q-\frac{9}{4})(4q^2-117q+81)\geqslant 0$

 

Dấu "=" xảy ra khi $(a,b,c)\sim (1,1,1);(0,\frac{3}{2},\frac{3}{2})$ và các hoán vị


Bài viết đã được chỉnh sửa nội dung bởi Minhnguyenthe333: 27-11-2016 - 20:21


#4
quanguefa

quanguefa

    Thiếu úy

  • Thành viên
  • 596 Bài viết

$VT=\frac{abc(a^2b+b^2c+c^2a)+9(ab+bc+ca)+3abc+3(a^2b^2+b^2c^2+c^2a^2)}{a^2b^2c^2+9abc+9(ab+bc+ca)+27}$

+Áp dụng bđt Cauchy: $\sum_{cyclic} (a^3+a^2b+ab^2)\geqslant 3(a^2b+b^2+c^2a)\iff a^2+b^2+c^2\geqslant a^2b+b^2c+c^2a$

 

+Đổi biến $p,q,r$ thì ta được: $VT\leqslant \frac{3q^2-2qr+9q}{r^2+9r+9q+27}$

Do đó ta chỉ cần chứng minh $\frac{3q^2-2qr+9q}{r^2+9r+9q+27}\leqslant \frac{3}{4}\iff 3r^2+r(8q+27)-(12q^2+9q-81)\geqslant 0$ $(*)$

Ta xét 2 trường hợp:
$q\leqslant \frac{9}{4}:$ Theo Schur bậc 1 thì $r\geqslant \max\{0,\frac{p(4q-p^2)}{9}\}=\max\{0,\frac{4q-9}{3}\}=0$

Thế thì $(*)\geqslant (q+3)(\frac{9}{4}-q)\geqslant 0$

$\frac{9}{4}\leqslant q\leqslant 3:$ Theo Schur bậc 2 thì $r\geqslant \frac{(4q-p^2)(p^2-q)}{6p}=\frac{(4q-9)(9-q)}{18}$

Do đó $(*)\geqslant (q-3)(q-\frac{9}{4})(4q^2-117q+81)\geqslant 0$

 

Dấu "=" xảy ra khi $(a,b,c)\sim (1,1,1);(0,\frac{3}{2},\frac{3}{2})$ và các hoán vị

cho mình hỏi ngu là đi thi không có máy tính thì khúc sau xử lý nổi không :3 


Xem topic "Chuyên đề các bài Toán lãi suất Casio" tại đây

 

:like Visit my facebook





1 người đang xem chủ đề

0 thành viên, 1 khách, 0 thành viên ẩn danh